FightingWolferine
FightingWolferine
  • 92
  • 52 947

Video

Can an irrational number be raised to an irrational power be rational?
zhlédnutí 33Před 7 měsíci
Can an irrational number be raised to an irrational power be rational? Guiton Sketch by Kevin MacLeod is licensed under a Creative Commons Attribution 4.0 license. creativecommons.org/licenses/by/4.0/ Source: incompetech.com/music/royalty-free/index.html?isrc=USUAN1100473 Artist: incompetech.com/
Can you solve this sequence problem from IMO 1986 Short List
zhlédnutí 121Před 7 měsíci
[IMO 1986 Short List] a_0=a_1=1, a_n = 7a_{n-1}-a_{n-2}-2, prove that all a_n are perfect squares. Using conventional approach will be too tedious. You need help from your instinct.
Can you solve this Olympiad problem from Austria 2005?
zhlédnutí 1,6KPřed 7 měsíci
If we know a b c=2, a^2 b^2 c^2=6, a^3 b^3 c^3=14, prove that |s_n^2-s_{n-1}s_{n 1}|=8
Find the minimum value of sqrt(x^2+y^2) if 5x+12y=60
zhlédnutí 414Před 7 měsíci
Introducing a form of Cauchy's inequality that is efficient for solving this type of problem
[IMO-1981] Find f(3, 2023) when f(x, y) satisfy certain rules
zhlédnutí 102Před 8 měsíci
Find f(3, 2023) if f(0,y)=y 1, f(x 1,0)=f(x,1), f(x 1,y 1)=f(x,f(x 1,y)). This is a question from IMO-1981 and the approach is now widely used in a type of computer algorithm.
Find the value of an expression of a, b, c, when they are roots of P(x)=x^3-2007x+2002
zhlédnutí 179Před 8 měsíci
A junior math Olympiad problem from Ireland and using Vieta theorem is not the best solution.
What is the probability of answering this correctly?
zhlédnutí 41Před 8 měsíci
Let's try to use game theory looking at a controversial logic problem.
If ab+bc+ac=0, calculate a^2/(a^2+2bc)+b^2/(b^2+2ac)+c^2/(c^2+2ab)
zhlédnutí 198Před 8 měsíci
If (a b c)^2=a^2 b^2 c^2, calculate a^2/(a^2 2bc) b^2/(b^2 2ac) c^2/(c^2 2ab). The key for the calculation is to preserve pieces that are symmetric among a, b, and c.
Find an irrational number a, such that a^2+2a and a^3-6a are both rational
zhlédnutí 610Před 8 měsíci
Find an irrational number a, such that a^2 2a and a^3-6a are both rational. This problem checks the fact that rational numbers are closed with regard to the operation of summation, subtraction, multiplication, and division.
Find the real roots of equation (97-x)^(1/4)+x^(1/4)=5
zhlédnutí 361Před 8 měsíci
Find the real roots of equation (97-x)^(1/4) x^(1/4)=5. The solution uses the property of homogeneous polynomials.
How to solve a cubic equation?
zhlédnutí 148Před 8 měsíci
A brief introduction to how to find the closed form solution for a generic cubic equation
What can you get from 1/a+1/b+1/c=1/(a+b+c)
zhlédnutí 105Před 9 měsíci
Another algebra identity proof from 1/a 1/b 1/c=1/(a b c). This is a typical category of junior math Olympiad problem: derive a relationship among a, b, c from a simple expression and use that to simplify a more complicated expression.
Find a six-digit number x, such that 2x, 3x, 4x, 5x, 6x all contain the same digits in x.
zhlédnutí 58Před 9 měsíci
Find a six-digit number x, such that 2x, 3x, 4x, 5x, 6x all contain the same digits in x. This is a one-of-a-kind puzzle that tests mathematical logics.
Prove that 8p-1 is a composite number when p and 8p+1 are both prime
zhlédnutí 671Před 9 měsíci
Prove that 8p-1 is a composite number when p and 8p 1 are both prime; Similarly, prove that 8p^2-1 is a prime number when p and 8p^2 1 are both prime.
If x+y+z=0, prove that x^3+y^3+z^3=3xyz
zhlédnutí 180Před 9 měsíci
If x y z=0, prove that x^3 y^3 z^3=3xyz
Find a_2023 if a_{n+2}=(1+a_{n+1})/a_n
zhlédnutí 457Před 9 měsíci
Find a_2023 if a_{n 2}=(1 a_{n 1})/a_n
Solve this equation system
zhlédnutí 66Před 9 měsíci
Solve this equation system
When a+b+c=0, calculate the value of this expression
zhlédnutí 287Před 9 měsíci
When a b c=0, calculate the value of this expression
Prove that 1/2+1/3+..+1/n is not an integer
zhlédnutí 587Před 9 měsíci
Prove that 1/2 1/3 .. 1/n is not an integer
Calculate sum{sqrt[1+i^(-2)+(i+1)^(-2)]}
zhlédnutí 369Před 9 měsíci
Calculate sum{sqrt[1 i^(-2) (i 1)^(-2)]}
Prove that the integer part of (2+sqrt3)^n is odd
zhlédnutí 321Před 9 měsíci
Prove that the integer part of (2 sqrt3)^n is odd
[Corrected] Solve equation: x^2+[x]=3
zhlédnutí 138Před 9 měsíci
[Corrected] Solve equation: x^2 [x]=3
Prove [x]+[x+1/n]+..+[x+(n-1)/n]=[nx]
zhlédnutí 383Před 9 měsíci
Prove [x] [x 1/n] .. [x (n-1)/n]=[nx]
Solve equation: x^2+[x]=3
zhlédnutí 2,7KPřed 9 měsíci
Solve equation: x^2 [x]=3
Solve equation: x^3+[x]=3
zhlédnutí 159Před 9 měsíci
Solve equation: x^3 [x]=3
Factorize (ab-1)(a+b)^2+1
zhlédnutí 133Před 9 měsíci
Factorize (ab-1)(a b)^2 1
Solve for distinct natural number of x,y, such that x^y = y^x
zhlédnutí 522Před 10 měsíci
Solve for distinct natural number of x,y, such that x^y = y^x
Solve x in sqrt(x+2sqrt(x+...+2sqrt(x+2sqrt(3x))))=x
zhlédnutí 206Před 10 měsíci
Solve x in sqrt(x 2sqrt(x ... 2sqrt(x 2sqrt(3x))))=x
Prove 1/(n+1) + 1/(n+2) + ... + 1/(3n+1) is between 1 and 2
zhlédnutí 728Před 10 měsíci
Prove 1/(n 1) 1/(n 2) ... 1/(3n 1) is between 1 and 2

Komentáře

  • @sunilsaini-rd9es
    @sunilsaini-rd9es Před 3 měsíci

    love from india

  • @_i_am_yours_tanmay_
    @_i_am_yours_tanmay_ Před 6 měsíci

    Can you please derive a formula for a^n + b^n and a^n - b^n where n are natural numbers

  • @bobbybannerjee5156
    @bobbybannerjee5156 Před 7 měsíci

    Sorry I didn't understand your logic.

    • @FightingWolferine
      @FightingWolferine Před 7 měsíci

      If sqrt2^sqrt2 is rational, the statement is already true; if sqrt2^sqrt2 is irrational, (sqrt2^sqrt2)^sqrt2 is rational, which shows the statement is true

    • @bobbybannerjee5156
      @bobbybannerjee5156 Před 7 měsíci

      @@FightingWolferine You said "if sqrt2^sqrt2 is irrational, (sqrt2^sqrt2)^sqrt2 is rational, which shows the statement is true". How?

    • @FightingWolferine
      @FightingWolferine Před 7 měsíci

      @@bobbybannerjee5156 if (sqrt2)^(sqrt2) is irrational, and we know sqrt2 is irrational, and notice that [(sqrt^2)^(sqrt2)]^sqrt2=(sqrt2)^(sqrt2*sqrt2)=(sqrt2)^2=2 is rational. So the statement is true.

  • @justarandomnerd3360
    @justarandomnerd3360 Před 7 měsíci

    no i cant

  • @presleynickel9527
    @presleynickel9527 Před 7 měsíci

    amazing proof 🙏🏼🙏🏼

  • @anishdeshpande3495
    @anishdeshpande3495 Před 7 měsíci

    Can’t you just use calculus to solve this easier?

    • @FightingWolferine
      @FightingWolferine Před 7 měsíci

      Sure, yet the goal of this video is not to teach calculus or college math.

  • @PantheraLeo04
    @PantheraLeo04 Před 8 měsíci

    This was a really great video, you did a great job breaking down the problem. But I have one question. If we hold x constant at some value n like you did in the video, would the non-recursive equation for f(x,y) always involve 2[n]y where [n] is the nth hyperoperation? (Except n=0, which would just be y[0] since the 0th hyperoperation only takes one input)

    • @FightingWolferine
      @FightingWolferine Před 8 měsíci

      Apparently yes. Thanks for bringing this up. I previously didn't know the fact that this is actually the famous two-argument Ackermann function: en.wikipedia.org/wiki/Ackermann_function

  • @vks_quily9157
    @vks_quily9157 Před 8 měsíci

    Nice 👍

  • @Dexous2232
    @Dexous2232 Před 8 měsíci

    Bro, in school these lines were alien to me. They still are to this day. I just can't fucking understand how your brains work

  • @eddietime1811
    @eddietime1811 Před 8 měsíci

    beautiful!

  • @brickie9816
    @brickie9816 Před 8 měsíci

    Extremely clever solution to a problem that seemed impossible for me. One of the best 5 minutes spent in my life. Thank you very much!

  • @marouaniAymen
    @marouaniAymen Před 8 měsíci

    I liked the video, can we compute sine, cosine and tangent by hand also as in your series ? Thanks again.

    • @FightingWolferine
      @FightingWolferine Před 8 měsíci

      I can only think of using Taylor's series, where: sin(x)=x-x^3/3!+x^5/5!-x^7/7!+...I can make a video for that;-)

    • @marouaniAymen
      @marouaniAymen Před 8 měsíci

      @@FightingWolferine I agree, I think it's basically how to compute fractions by hand, with this a playlist on how to navigate without a computer may be plausible.

  • @brickie9816
    @brickie9816 Před 9 měsíci

    Very nice problem. Thank you for the video

  • @zhichaohuang8310
    @zhichaohuang8310 Před 9 měsíci

    Man, this is tedious 😂

    • @FightingWolferine
      @FightingWolferine Před 9 měsíci

      Right, that’s why it’s not as popular/famous as the one for quadratic equations;)

  • @abhishekraj2336
    @abhishekraj2336 Před 9 měsíci

    So, according to you conditions are -2.56 < x < -2.31 and 1.31 < x < 1.56. You got x = sqrt(2) = 1.414 ✔ and -sqrt(5) = -2.236 ✘, -2.236 is clearly not lying in the range of x you got.

    • @FightingWolferine
      @FightingWolferine Před 9 měsíci

      See the other video linked. The definition of [x] here is not the floor of x. [-2.236]=-2 not -3 with this notation. We take the definition of integer part of x as the floor if x is positive but the ceiling if x is negative. An error in the video was that by this definition {x} is between -1 and 0 when x is negative and this was corrected in the linked video. The final solution is still correct.

  • @autumn4442
    @autumn4442 Před 9 měsíci

    The way I figured out the first problem is this: In order for it to be false, it must be impossible for all 3 to be prime. Thus, I can instead prove that if 8p+1 and 8p-1 are prime, p is not prime. All even numbers have no less than one 2 in their prime factorization, and no odd numbers ever have a 2 in their prime factorization. Therefore, 8p must be even, making 8p+1 and 8p-1 odd. All primes>2 are odd, and every third odd number is a multiple of 3. Therefore, in order for two consecutive odd numbers to both be prime (in this case, 8p+1 and 8p-1), the odd number before it (in this case, 8p-3), must be a multiple of 3. This means that 8p must be a multiple of 3, since 8p-3+3 is 8p, and any multiple of 3, plus 3, is still a multiple of 3. If 8p is a multiple of 3, then p must be a multiple of 3, since 8p/8 doesn't cancel out the 3 in 8p's prime factorization. Because p is a multiple of 3, the only way for it to be prime is if p=3. Checking that one case shows that 8p+1=25, which is not prime. If 8p+1 and 8p-1 are both prime, then it is impossible for p to be prime. Therefore, if p is prime, it is impossible for both 8p+1 and 8p-1 to be prime. If p and 8p+1 are prime, 8p-1 is not prime. I think it's fundamentally the same proof as in the video, just expressed differently, but I don't know for sure since I had trouble following the video.

    • @FightingWolferine
      @FightingWolferine Před 9 měsíci

      It’s essentially the same but it’s easier to describe using with congruence modulo concept: basically, discuss the remainder of p, 8p-1, and 8p+1 divided by 3. In order for all three to be prime, none of the reminders should be 0.

    • @autumn4442
      @autumn4442 Před 9 měsíci

      @@FightingWolferine Yes, but why check with modulo 3? If you already know it works as a solution, it's obvious, but why would someone new to the problem check with modulo 3 instead of, say, modulo 5? While what you said does indeed work as a proof, it still leaves one question unanswered: why did you do it that specific way? My method is much less concise, but I believe it's much easier to follow *why* I'm doing what I'm doing.

    • @FightingWolferine
      @FightingWolferine Před 9 měsíci

      You can list out the first several prime numbers of p and it is quite obvious that in many cases 8p+1 is a multiple of 3. Then you can try the idea to examine the remainder of them divided by 3.

    • @FightingWolferine
      @FightingWolferine Před 9 měsíci

      Another way to think about this is: 8p+1, 8p, 8p-1 are three consecutive integers. One and only one of them can be divided by 3.

    • @autumn4442
      @autumn4442 Před 9 měsíci

      @@FightingWolferine Ah, writing them out as 3 consecutive numbers makes sense, though the first reason you gave is pretty much "just brute force it lol." Though, that makes the explanation a lot easier. Half of the difficulty with mine is just proving that 8p is a multiple of 3 if 8p+1 and 8p-1 are prime. I can't believe I missed that obvious explanation. I think with that in mind it's far more elegant to prove that 8p is a multiple of 3 the way you said, and then just use the logic I gave to prove that if 8p is a multiple of 3, then p is not prime, no modulo needed.

  • @zhichaohuang8310
    @zhichaohuang8310 Před 9 měsíci

    p and 8p-1 are not multiples of 3 is not enough to prove that they are prime numbers

    • @FightingWolferine
      @FightingWolferine Před 9 měsíci

      No it is not. But the other direction is true: if p and 8p-1 are prime, they cannot be multiplies of 3.

  • @aswinkumar4051
    @aswinkumar4051 Před 9 měsíci

    Nice video...i like to work maths problems...so please upload more problems like this

  • @praveenashomelife8127
    @praveenashomelife8127 Před 9 měsíci

    I thought of another way By using the identity that when a+b+c=0 Then a³+b³+c³= 3abc As it was given a³+b³+c³= 0 We can directly say abc=0 Now if we consider any one if them as zero we would get that each of the other two numbers will be negative to each other or three of them will be zero In some conditions guess many not be right

    • @FightingWolferine
      @FightingWolferine Před 9 měsíci

      Right, you need to use the factorization of a^3+b^3+c^3-3abc, which is not obvious if you don’t already know the identity.

  • @a_minor
    @a_minor Před 9 měsíci

    The institute i go to only ever taught me the formula, never the proof. I really appreciate this video.

  • @GuruprakashAcademy
    @GuruprakashAcademy Před 9 měsíci

    -sqrt5 does not lies in the range

    • @FightingWolferine
      @FightingWolferine Před 9 měsíci

      @GuruprakashAcademy please see my other comments. [x] is not the floor notation. It represents the integer part of x, which equals -2 for -2.56.

    • @GuruprakashAcademy
      @GuruprakashAcademy Před 9 měsíci

      @@FightingWolferine I think [] denotes the greatest integer function i.e., [-2.56] =-3

    • @FightingWolferine
      @FightingWolferine Před 9 měsíci

      @@GuruprakashAcademy Mathematical notations can be complicated. What I learned at school and referred to in this video is in alignment with the explanation in Wikipedia: en.wikipedia.org/wiki/Floor_and_ceiling_functions. But historically, the notation could be different. Anyways, hope this clarifies the misunderstanding.

    • @GuruprakashAcademy
      @GuruprakashAcademy Před 9 měsíci

      @@FightingWolferine Thanks. But with this notation, I think we cannot write x = integer part + fractional part... Just check for any negative numbers.

    • @FightingWolferine
      @FightingWolferine Před 9 měsíci

      @@GuruprakashAcademy That's a good question. The fractional part (or decimal part) of x can actually have different definitions: en.wikipedia.org/wiki/Fractional_part. The odd function definition actually persists the relationship of x=[x] + {x}, however, {x} is between (-1,0] when x<0. So the solution process in the video was wrong to claim 0 <= {x} < 1 universally. The solution was right for x>0. When x<0, -1 < {x} <= 0, thus -1 < x^2 + x - 3 < 0, this leads to -2.3 < x < -2 (only take the negative range), which still leads to [x] = -2. So the final solution is still the same. But thanks for pointing out the issue.

  • @GuruprakashAcademy
    @GuruprakashAcademy Před 9 měsíci

    [-2.56] = -3

    • @FightingWolferine
      @FightingWolferine Před 9 měsíci

      Let me clarify the notation: Floor(-2.56)=⌊-2.56⌋=-3, ceil(-2.56)=⌈-2.56⌉=-2, integer part of -2.56 = [-2.56]=-2.

  • @archangecamilien1879
    @archangecamilien1879 Před 9 měsíci

    x=sqrt(2), perhaps...floor is 1, and square is 2, so 2+1=3...

    • @FightingWolferine
      @FightingWolferine Před 9 měsíci

      Right, but this solution is not complete. Need to capture all solutions.

    • @archangecamilien1879
      @archangecamilien1879 Před 9 měsíci

      Ah, ok...

    • @archangecamilien1879
      @archangecamilien1879 Před 9 měsíci

      I think those are two only solutions, though I didn't check to see...x^2 = 3 - floor[x], which means x^2 is an integer, so x if of the form +-sqrt(n) for some integer n (I mean, lol, I assume these are supposed to be real numbers)...so, for positive x, x^2 = 3 - floor [x], just check all the squares for x^2, there are finitely many...very few, actually, lol...

    • @archangecamilien1879
      @archangecamilien1879 Před 9 měsíci

      As for negative x, then it's the equivalent of an equation using y=-x, which would give us y^2 = 3 + floor [y], y positive...now, y^2 > 2y>3+y>=3+floor[y], if y>3, which is obvious...I mean, I could prove it too, lol...remembering that y>=floor[y], since y is positive...so that means we need only check y up to 3, no need to go further...which means that we need to descend only to -3 for x negative...so, lol, since x^2 is an integer, it's a short and finite search, I suspect sqrt(2) and -sqrt(6) are the only solutions, but maybe I'm wrong...

    • @archangecamilien1879
      @archangecamilien1879 Před 9 měsíci

      Basically, just try all x^2 from 0 to 9 or so (3^2 = (-3)^2 = 9), etc...

  • @arekkrolak6320
    @arekkrolak6320 Před 9 měsíci

    first thing you notice is that [x] is only defined for real numbers, so any solution must be real, and both [x] and 3 is integer, so for the equation to be valid x^2 also must be integer, no need to check any rational values that are not integers like 2.31; you can just make a table starting from x=0 and going in both directions (positive and negative) assigning all the values that give integer square and check where the result is 3, once the result in any direction goes above 3 it will always be greater so no need to check further

    • @FightingWolferine
      @FightingWolferine Před 9 měsíci

      The downside of your approach is if the right hand side is a much greater number (e.g. A=1000], then iterating A-[x] will be tedious.

  • @mkmishra.1997
    @mkmishra.1997 Před 10 měsíci

    Great solution!

  • @moeberry8226
    @moeberry8226 Před 10 měsíci

    Your video is wrong the floor of -2.56-2.31 is not -2 but rather -3.

    • @FightingWolferine
      @FightingWolferine Před 10 měsíci

      The integer part of x is defined as the floor of x when x>0, the ceiling of x when x<0. The integer part of -2.56 is -2, not -3.

    • @moeberry8226
      @moeberry8226 Před 10 měsíci

      @@FightingWolferine that’s true the integer part is defined that way but the notation in your video looks like x^2 + the floor of x.

    • @sa0o923
      @sa0o923 Před 10 měsíci

      ​@@moeberry8226 your comment is wrong. The floor notation is square bracket with top part open.

    • @yassinezanned9837
      @yassinezanned9837 Před 10 měsíci

      Yes you're actually right, the floor of that interval is -3

    • @yassinezanned9837
      @yassinezanned9837 Před 10 měsíci

      The other solution is -√6 not -√5

  • @osmosisjohns5650
    @osmosisjohns5650 Před 10 měsíci

    Good video thank you; )

  • @Goodlandchugs
    @Goodlandchugs Před 10 měsíci

    2^4=4^2 ; x=2 y=4

    • @FightingWolferine
      @FightingWolferine Před 10 měsíci

      Right, for every solution obtained under y>x, you can swap x and y and get the counterpart solution under y<x.

    • @FightingWolferine
      @FightingWolferine Před 10 měsíci

      Oh, misunderstood the comment. Yes, it is not very hard to guess ONE solution but it takes some interesting thoughts to prove that’s the ONLY solution to the problem.

  • @El_Girasol_Fachero
    @El_Girasol_Fachero Před 10 měsíci

    Good! 👍🏻✨

  • @_i_am_yours_tanmay_
    @_i_am_yours_tanmay_ Před 10 měsíci

    What will be the value of 1^i

  • @alejandroduque772
    @alejandroduque772 Před 10 měsíci

    Interesting way to solve it. I considered the real function f=(1+1/x)^x (which is clearly positive). Taking logs and deriving, we can see that the derivatice is positive iff log(1+1/x)-1/(1+x) is positive. We can derive this second function to verify it is decreasing for positive reals with lower bound zero and with upper bound ln2-1/2 (which is positive). All in all, we get that f is increasing and taking n to infinity we can see that an upper bound is e<3. The lower bound is taken from n=2

    • @FightingWolferine
      @FightingWolferine Před 10 měsíci

      The last part is wrong: you are not supposed to use the conclusion that the upper bound is e<3. The convergence is based on the fact that this function is monotonously increasing and has an upper bound and we denote lim(1+1/x)^x=e. So the upper bound needs to be proved separately.

    • @alejandroduque772
      @alejandroduque772 Před 10 měsíci

      @@FightingWolferine Are you saying that stating e<3 is sort of a circular reasoning for the boundedness of the sequence (?)

    • @FightingWolferine
      @FightingWolferine Před 10 měsíci

      Yes, since that’s how e is defined

  • @samuelgomez3614
    @samuelgomez3614 Před 10 měsíci

    The conclusion is wrong if you're trying to show this for all naturals. You need <= on the left inequality. Otherwise let n=1 (so k=1, since k=n) and you're claiming that 2<2.

  • @areyousrsrightneowbro
    @areyousrsrightneowbro Před 10 měsíci

    As n->0 it approaches 1 which is less than 2

    • @FightingWolferine
      @FightingWolferine Před 10 měsíci

      Right. Should have stated that here n is a natural number

  • @DarkIzo
    @DarkIzo Před 10 měsíci

    quick mafs

  • @KarimFilali413
    @KarimFilali413 Před 10 měsíci

    I don't understand why x + y | x^k + y^k.

    • @FightingWolferine
      @FightingWolferine Před 10 měsíci

      Consider f(x,y)=x^k+y^k, because f(x,-x)=0, thus y=-x is a root to f(x,y)=0. Thus x+y is a factor of f(x,y). This is similar to e.g.: f(x)=x^3-x, f(1)=0, thus x-1 is a factor of f(x)

  • @marouaniAymen
    @marouaniAymen Před 10 měsíci

    Beautiful, I hope that for a video related to those closed sums we'll understand Bernouilli Coefficients.

  • @normanagudelomejia
    @normanagudelomejia Před 10 měsíci

    Recuerdo cuando abordé el mismo problema y llegué a la misma solución por otros medios deductivos. Para aquel momento tenia 15 años , aun estaba en el colegio y nunca había escrito una sumatoria.

  • @marouaniAymen
    @marouaniAymen Před 10 měsíci

    Thanks for the interesting video, this result reminds me the relationship between the sum of cubes and the sum of integers for their closed forms, but I think for infinite boundary (n is infinite) we have to recall other concepts, perhaps equivalence in limits or radius of convergence.

  • @suryanshnayak7538
    @suryanshnayak7538 Před 10 měsíci

    wont say its tricky or anything its just like if u have seen this type of problems u know what to substitute ... either way once we substitute the range always lies between whole underroot a power 2 + b power 2 .. this is helpful if u dont want to find the roots

    • @FightingWolferine
      @FightingWolferine Před 10 měsíci

      This substitution simplifies the condition: from a relationship between x and y to only on the radius r (essentially from Cartesian to polar). It’s useful in solving range problems.

  • @anshumanagrawal346
    @anshumanagrawal346 Před 10 měsíci

    How do you know (-1)^q is -1 ?

  • @sanskarsapkal-._.-
    @sanskarsapkal-._.- Před 10 měsíci

    Very interesting problem sir❤✌️

  • @dinosoeren
    @dinosoeren Před 10 měsíci

    very cool!

  • @vks_quily9157
    @vks_quily9157 Před 10 měsíci

    Nice problem 😄

  • @Chitin-qy7xy
    @Chitin-qy7xy Před 11 měsíci

    U r so smart

  • @fotisp6293
    @fotisp6293 Před 11 měsíci

    The fact that p divides a² (and hence a, since p is a prime) (and similar for b² and b), does not follow from the fact that gcd(a,b)=1 (and later gcd(b,k)=1)but rather from the fact that in the equation a²=pb², the right hand side is divisible by p, and so the left hand side must also be divisible by p (and similarly for b²=k²p) Appart from this little detail, excellent video!

    • @tiripoulain
      @tiripoulain Před 11 měsíci

      Also the fact that p|a and p|b does not imply that gcd(a,b)=p, but merely that p|gcd(a,b).

    • @FightingWolferine
      @FightingWolferine Před 11 měsíci

      Right, mentioned in the narration but didn’t add that in the slide;)

    • @FightingWolferine
      @FightingWolferine Před 11 měsíci

      It implies gcd(a,b) is at least p

    • @tiripoulain
      @tiripoulain Před 11 měsíci

      @@FightingWolferine Yes.

  • @Lankarix
    @Lankarix Před 11 měsíci

    Thats cool

  • @itismethatguy
    @itismethatguy Před 11 měsíci

    Cool, +1 sub